TABE Reading Practice Test: Level D

This free practice test covers Level D of TABE Reading. It’s more challenging than the prior ones, just one step below the advanced level. These passages reflect situations from work and life in order to test your word-meaning skills, critical-thinking skills, and more. Continue your test prep now with our TABE Reading Level D practice questions!

Congratulations - you have completed .

You scored %%SCORE%% out of %%TOTAL%%.

Your performance has been rated as %%RATING%%


Your answers are highlighted below.
Question 1
Violent Ways

(1) The modern world has a bit of an identity crisis when it comes to their love of sports. On one end of the spectrum, we worry about violence in sports. We’re concerned about head injuries in football. We worry that our children are focusing so early on one sport that they are causing their body harm. We argue that we need to make changes to our current systems to protect our athletes. At the same time, though, violent sports like mixed martial arts have never been more popular. How do we explain such a split personality in how we view athletes? Why are we worried about a quarterback getting hit too hard but cheering when a fighter lifelessly collapses to the mat like a dead tree?

(2) The answer to these questions is at the same time simple and complicated. The simple answer is that a sport like mixed martial arts is intended to be violent; the sport involves two people getting into a ring and fighting. Football, on the other hand, centers around moving a ball from one zone to the next. To argue that football is not about violence would be extremely naïve, though. That’s where the problem gets complicated. If we remove violence from football, what’s left?

(3) The more complicated and depressing reason why we can’t seem to figure out whether we want violence or not is because it plays to our human nature. The football arena is the new Colosseum, and the players are the new gladiators. One of the most powerful human instinct is the instinct of survival. Naturally, then, we are entertained when people are in peril because we are instinctively drawn to their fight for survival. This doesn’t make us evil; we don’t want to see people get hurt. We do, however, want to see people get hit.

(4) The problem is that there is no definitive solution to violence in sports. We want it and don’t want it at the same time. It seems our culture has decided to allow it in certain areas where we think it belongs, like mixed martial arts, but condemning it where we think it doesn’t, like football. Football fans worry about what’s going to happen to the sport in the next few years as reports of brain injuries continue to surface. Most likely, though, nothing will change. We will sit around our dinner table and argue that something needs to be done, then head to the Colosseum and cheer for every big hit.
 

Read this sentence from the passage.

The football arena is the new Colosseum, and the players are the new gladiators.

Which of these best explains the meaning of the phrase “the players are the new gladiators”?

A
Football is an outdated game that needs to fall just like the empire of Rome fell.
B
Football players are violent and want to hurt each other.
C
People like to see football players hit each other in the same way that people loved to watch gladiators kill themselves in ancient Rome.
D
The more we watch football players, the more we give in to our instincts and cause harm to our society.
Question 1 Explanation: 
The author alludes to ancient Rome in this quote in order to try to explain our obsession with violence. Answer choice (C) best captures this meaning and tone. The author isn’t condemning football player (B), or calling for the end of football (A) (D), she is instead offering an insight into why Americans like it.
Question 2
Violent Ways

(1) The modern world has a bit of an identity crisis when it comes to their love of sports. On one end of the spectrum, we worry about violence in sports. We’re concerned about head injuries in football. We worry that our children are focusing so early on one sport that they are causing their body harm. We argue that we need to make changes to our current systems to protect our athletes. At the same time, though, violent sports like mixed martial arts have never been more popular. How do we explain such a split personality in how we view athletes? Why are we worried about a quarterback getting hit too hard but cheering when a fighter lifelessly collapses to the mat like a dead tree?

(2) The answer to these questions is at the same time simple and complicated. The simple answer is that a sport like mixed martial arts is intended to be violent; the sport involves two people getting into a ring and fighting. Football, on the other hand, centers around moving a ball from one zone to the next. To argue that football is not about violence would be extremely naïve, though. That’s where the problem gets complicated. If we remove violence from football, what’s left?

(3) The more complicated and depressing reason why we can’t seem to figure out whether we want violence or not is because it plays to our human nature. The football arena is the new Colosseum, and the players are the new gladiators. One of the most powerful human instinct is the instinct of survival. Naturally, then, we are entertained when people are in peril because we are instinctively drawn to their fight for survival. This doesn’t make us evil; we don’t want to see people get hurt. We do, however, want to see people get hit.

(4) The problem is that there is no definitive solution to violence in sports. We want it and don’t want it at the same time. It seems our culture has decided to allow it in certain areas where we think it belongs, like mixed martial arts, but condemning it where we think it doesn’t, like football. Football fans worry about what’s going to happen to the sport in the next few years as reports of brain injuries continue to surface. Most likely, though, nothing will change. We will sit around our dinner table and argue that something needs to be done, then head to the Colosseum and cheer for every big hit.
 

How does Paragraph 2 contribute to the development of ideas in the passage?

A
It establishes the complexity of the topic the author is discussing.
B
It simplifies the topic to make it more palatable and digestible for the audience.
C
It presents a counter-argument that she will refute later in the article.
D
It defines the author’s thesis and sets up the rest of the article.
Question 2 Explanation: 
Answer choice (A) is correct because even though the author initially offers a simple answer to the question she posed in the previous paragraph, she also takes that answer apart and prepares the audience for the complexities she will detail throughout the rest of the argument. This isn’t necessarily a counter-argument (C), nor does it set up the author’s thesis (D). She presents her thesis in Paragraph 3.
Question 3
Violent Ways

(1) The modern world has a bit of an identity crisis when it comes to their love of sports. On one end of the spectrum, we worry about violence in sports. We’re concerned about head injuries in football. We worry that our children are focusing so early on one sport that they are causing their body harm. We argue that we need to make changes to our current systems to protect our athletes. At the same time, though, violent sports like mixed martial arts have never been more popular. How do we explain such a split personality in how we view athletes? Why are we worried about a quarterback getting hit too hard but cheering when a fighter lifelessly collapses to the mat like a dead tree?

(2) The answer to these questions is at the same time simple and complicated. The simple answer is that a sport like mixed martial arts is intended to be violent; the sport involves two people getting into a ring and fighting. Football, on the other hand, centers around moving a ball from one zone to the next. To argue that football is not about violence would be extremely naïve, though. That’s where the problem gets complicated. If we remove violence from football, what’s left?

(3) The more complicated and depressing reason why we can’t seem to figure out whether we want violence or not is because it plays to our human nature. The football arena is the new Colosseum, and the players are the new gladiators. One of the most powerful human instinct is the instinct of survival. Naturally, then, we are entertained when people are in peril because we are instinctively drawn to their fight for survival. This doesn’t make us evil; we don’t want to see people get hurt. We do, however, want to see people get hit.

(4) The problem is that there is no definitive solution to violence in sports. We want it and don’t want it at the same time. It seems our culture has decided to allow it in certain areas where we think it belongs, like mixed martial arts, but condemning it where we think it doesn’t, like football. Football fans worry about what’s going to happen to the sport in the next few years as reports of brain injuries continue to surface. Most likely, though, nothing will change. We will sit around our dinner table and argue that something needs to be done, then head to the Colosseum and cheer for every big hit.
 

Read the sentence from Paragraph 1.

The modern world has a bit of an identity crisis when it comes to their love of sports.

Based on the context of the passage, what is the problem (if any) with this quote?

A
A culture cannot have an “identity crisis” because it is not a living thing.
B
The author introduces the “world” in this quote, but only discusses American culture throughout the article.
C
The word “bit” is too informal for the article and distracts the reader.
D
There is no flaw in the logic of the statement.
Question 3 Explanation: 
The author introduces the “world” in this quote, but then discusses mainly football and mixed martial arts, which are both mostly American sports. If the author intends to discuss the state of the world in the article, then she has alienated a lot of her audience by only discussing things that Americans can relate to.
Question 4
Violent Ways

(1) The modern world has a bit of an identity crisis when it comes to their love of sports. On one end of the spectrum, we worry about violence in sports. We’re concerned about head injuries in football. We worry that our children are focusing so early on one sport that they are causing their body harm. We argue that we need to make changes to our current systems to protect our athletes. At the same time, though, violent sports like mixed martial arts have never been more popular. How do we explain such a split personality in how we view athletes? Why are we worried about a quarterback getting hit too hard but cheering when a fighter lifelessly collapses to the mat like a dead tree?

(2) The answer to these questions is at the same time simple and complicated. The simple answer is that a sport like mixed martial arts is intended to be violent; the sport involves two people getting into a ring and fighting. Football, on the other hand, centers around moving a ball from one zone to the next. To argue that football is not about violence would be extremely naïve, though. That’s where the problem gets complicated. If we remove violence from football, what’s left?

(3) The more complicated and depressing reason why we can’t seem to figure out whether we want violence or not is because it plays to our human nature. The football arena is the new Colosseum, and the players are the new gladiators. One of the most powerful human instinct is the instinct of survival. Naturally, then, we are entertained when people are in peril because we are instinctively drawn to their fight for survival. This doesn’t make us evil; we don’t want to see people get hurt. We do, however, want to see people get hit.

(4) The problem is that there is no definitive solution to violence in sports. We want it and don’t want it at the same time. It seems our culture has decided to allow it in certain areas where we think it belongs, like mixed martial arts, but condemning it where we think it doesn’t, like football. Football fans worry about what’s going to happen to the sport in the next few years as reports of brain injuries continue to surface. Most likely, though, nothing will change. We will sit around our dinner table and argue that something needs to be done, then head to the Colosseum and cheer for every big hit.
 

Which sentence from the passage best expresses the author’s attitude toward violence in sports?

A
“The simple answer is that a sport like mixed martial arts is intended to be violent; the sport involves two people getting into a ring and fighting.”
B
“We worry that our children are focusing so early on one sport that they are causing their body harm.”
C
“Football fans worry about what’s going to happen to the sport in the next few years as reports of brain injuries continue to surface.”
D
“The more complicated and depressing reason why we can’t seem to figure out whether we want violence or not is because it plays to our human nature.”
Question 4 Explanation: 
Answer choice (D) is correct because it is the only quote that discusses the author’s explanation for the world’s obsession with violence. The other answer choices represent ideas that the author uses to build to her thesis, not directly express her attitude towards the subject.
Question 5
Violent Ways

(1) The modern world has a bit of an identity crisis when it comes to their love of sports. On one end of the spectrum, we worry about violence in sports. We’re concerned about head injuries in football. We worry that our children are focusing so early on one sport that they are causing their body harm. We argue that we need to make changes to our current systems to protect our athletes. At the same time, though, violent sports like mixed martial arts have never been more popular. How do we explain such a split personality in how we view athletes? Why are we worried about a quarterback getting hit too hard but cheering when a fighter lifelessly collapses to the mat like a dead tree?

(2) The answer to these questions is at the same time simple and complicated. The simple answer is that a sport like mixed martial arts is intended to be violent; the sport involves two people getting into a ring and fighting. Football, on the other hand, centers around moving a ball from one zone to the next. To argue that football is not about violence would be extremely naïve, though. That’s where the problem gets complicated. If we remove violence from football, what’s left?

(3) The more complicated and depressing reason why we can’t seem to figure out whether we want violence or not is because it plays to our human nature. The football arena is the new Colosseum, and the players are the new gladiators. One of the most powerful human instinct is the instinct of survival. Naturally, then, we are entertained when people are in peril because we are instinctively drawn to their fight for survival. This doesn’t make us evil; we don’t want to see people get hurt. We do, however, want to see people get hit.

(4) The problem is that there is no definitive solution to violence in sports. We want it and don’t want it at the same time. It seems our culture has decided to allow it in certain areas where we think it belongs, like mixed martial arts, but condemning it where we think it doesn’t, like football. Football fans worry about what’s going to happen to the sport in the next few years as reports of brain injuries continue to surface. Most likely, though, nothing will change. We will sit around our dinner table and argue that something needs to be done, then head to the Colosseum and cheer for every big hit.
 

Which of the following is an inference the audience can make about the author’s opinion in Paragraph 4?

A
When people think rationally, they don’t want violence, but when they are in the moment they enjoy it.
B
Until we stop supporting football, our country will never fix our problem with violence.
C
The popularity of violent sports like mixed martial arts are making it more difficult to curb violence in other sports.
D
The real way to fix our problem with violence in sports is to more closely regulate it so that our athletes are protected.
Question 5 Explanation: 
Answer choice (A) is correct because it is a more specific explanation of the author’s closing line. The author believes that people want to be against violence, but that they can’t help but be entertained by it when they see it. Answer choice (B) and (C) are not reflected in the article, and answer choice (D) goes against the opening sentence of Paragraph 4.
Question 6
Violent Ways

(1) The modern world has a bit of an identity crisis when it comes to their love of sports. On one end of the spectrum, we worry about violence in sports. We’re concerned about head injuries in football. We worry that our children are focusing so early on one sport that they are causing their body harm. We argue that we need to make changes to our current systems to protect our athletes. At the same time, though, violent sports like mixed martial arts have never been more popular. How do we explain such a split personality in how we view athletes? Why are we worried about a quarterback getting hit too hard but cheering when a fighter lifelessly collapses to the mat like a dead tree?

(2) The answer to these questions is at the same time simple and complicated. The simple answer is that a sport like mixed martial arts is intended to be violent; the sport involves two people getting into a ring and fighting. Football, on the other hand, centers around moving a ball from one zone to the next. To argue that football is not about violence would be extremely naïve, though. That’s where the problem gets complicated. If we remove violence from football, what’s left?

(3) The more complicated and depressing reason why we can’t seem to figure out whether we want violence or not is because it plays to our human nature. The football arena is the new Colosseum, and the players are the new gladiators. One of the most powerful human instinct is the instinct of survival. Naturally, then, we are entertained when people are in peril because we are instinctively drawn to their fight for survival. This doesn’t make us evil; we don’t want to see people get hurt. We do, however, want to see people get hit.

(4) The problem is that there is no definitive solution to violence in sports. We want it and don’t want it at the same time. It seems our culture has decided to allow it in certain areas where we think it belongs, like mixed martial arts, but condemning it where we think it doesn’t, like football. Football fans worry about what’s going to happen to the sport in the next few years as reports of brain injuries continue to surface. Most likely, though, nothing will change. We will sit around our dinner table and argue that something needs to be done, then head to the Colosseum and cheer for every big hit.
 

Which of the following statements best describes the structure of the author’s article?

A
The author introduces a complex problem, then offers several possible solutions.
B
The author presents two opposing sides to a complex argument and leaves it up to the reader decide with which side they agree.
C
The author provides a detailed, sequential strategy for solving a difficult problem.
D
The author introduces a complex problem, then explores it throughout the rest of the article.
Question 6 Explanation: 
The author offers plenty of opinions throughout the article, but never ventures into a real solution to the problem. Answer choice (D) is the only option that allows for this lack of a solution.
Question 7
Violent Ways

(1) The modern world has a bit of an identity crisis when it comes to their love of sports. On one end of the spectrum, we worry about violence in sports. We’re concerned about head injuries in football. We worry that our children are focusing so early on one sport that they are causing their body harm. We argue that we need to make changes to our current systems to protect our athletes. At the same time, though, violent sports like mixed martial arts have never been more popular. How do we explain such a split personality in how we view athletes? Why are we worried about a quarterback getting hit too hard but cheering when a fighter lifelessly collapses to the mat like a dead tree?

(2) The answer to these questions is at the same time simple and complicated. The simple answer is that a sport like mixed martial arts is intended to be violent; the sport involves two people getting into a ring and fighting. Football, on the other hand, centers around moving a ball from one zone to the next. To argue that football is not about violence would be extremely naïve, though. That’s where the problem gets complicated. If we remove violence from football, what’s left?

(3) The more complicated and depressing reason why we can’t seem to figure out whether we want violence or not is because it plays to our human nature. The football arena is the new Colosseum, and the players are the new gladiators. One of the most powerful human instinct is the instinct of survival. Naturally, then, we are entertained when people are in peril because we are instinctively drawn to their fight for survival. This doesn’t make us evil; we don’t want to see people get hurt. We do, however, want to see people get hit.

(4) The problem is that there is no definitive solution to violence in sports. We want it and don’t want it at the same time. It seems our culture has decided to allow it in certain areas where we think it belongs, like mixed martial arts, but condemning it where we think it doesn’t, like football. Football fans worry about what’s going to happen to the sport in the next few years as reports of brain injuries continue to surface. Most likely, though, nothing will change. We will sit around our dinner table and argue that something needs to be done, then head to the Colosseum and cheer for every big hit.
 

Part A

Which statement expresses a claim the author makes about society’s view of violence in sports?

A
Good people criticize violence in sports because they don’t want to see athletes get hurt.
B
Based on the popularity of different sports, people are not sure whether or not they like violence in sports.
C
The simple solution to the problem of too much violence in sports is for the government to design legislation that puts an end to it.
D
Over time, people will learn to prefer that sports don’t include any violent acts that could endanger an athlete.
Question 7 Explanation: 
The author spends a lot of time in the passage asserting that the problem is complicated and that there is no simple solution. Answer choice (B), on top of being accurate to the author’s point, is the only option that allows for this complexity.
Question 8
Violent Ways

(1) The modern world has a bit of an identity crisis when it comes to their love of sports. On one end of the spectrum, we worry about violence in sports. We’re concerned about head injuries in football. We worry that our children are focusing so early on one sport that they are causing their body harm. We argue that we need to make changes to our current systems to protect our athletes. At the same time, though, violent sports like mixed martial arts have never been more popular. How do we explain such a split personality in how we view athletes? Why are we worried about a quarterback getting hit too hard but cheering when a fighter lifelessly collapses to the mat like a dead tree?

(2) The answer to these questions is at the same time simple and complicated. The simple answer is that a sport like mixed martial arts is intended to be violent; the sport involves two people getting into a ring and fighting. Football, on the other hand, centers around moving a ball from one zone to the next. To argue that football is not about violence would be extremely naïve, though. That’s where the problem gets complicated. If we remove violence from football, what’s left?

(3) The more complicated and depressing reason why we can’t seem to figure out whether we want violence or not is because it plays to our human nature. The football arena is the new Colosseum, and the players are the new gladiators. One of the most powerful human instinct is the instinct of survival. Naturally, then, we are entertained when people are in peril because we are instinctively drawn to their fight for survival. This doesn’t make us evil; we don’t want to see people get hurt. We do, however, want to see people get hit.

(4) The problem is that there is no definitive solution to violence in sports. We want it and don’t want it at the same time. It seems our culture has decided to allow it in certain areas where we think it belongs, like mixed martial arts, but condemning it where we think it doesn’t, like football. Football fans worry about what’s going to happen to the sport in the next few years as reports of brain injuries continue to surface. Most likely, though, nothing will change. We will sit around our dinner table and argue that something needs to be done, then head to the Colosseum and cheer for every big hit.
 

Part B

Which sentence from the article best supports the answer to Part A?

A
“One of the most powerful human instinct is the instinct of survival.”
B
“Football, on the other hand, centers around moving a ball from one zone to the next.”
C
“At the same time, though, violent sports like mixed martial arts have never been more popular.”
D
“We will sit around our dinner table and argue that something needs to be done, then head to the Colosseum and cheer for every big hit.”
Question 8 Explanation: 
The closing sentence of the article sums up the hypocrisy that the author is pointing out in society very well.
Question 9
Violent Ways

(1) The modern world has a bit of an identity crisis when it comes to their love of sports. On one end of the spectrum, we worry about violence in sports. We’re concerned about head injuries in football. We worry that our children are focusing so early on one sport that they are causing their body harm. We argue that we need to make changes to our current systems to protect our athletes. At the same time, though, violent sports like mixed martial arts have never been more popular. How do we explain such a split personality in how we view athletes? Why are we worried about a quarterback getting hit too hard but cheering when a fighter lifelessly collapses to the mat like a dead tree?

(2) The answer to these questions is at the same time simple and complicated. The simple answer is that a sport like mixed martial arts is intended to be violent; the sport involves two people getting into a ring and fighting. Football, on the other hand, centers around moving a ball from one zone to the next. To argue that football is not about violence would be extremely naïve, though. That’s where the problem gets complicated. If we remove violence from football, what’s left?

(3) The more complicated and depressing reason why we can’t seem to figure out whether we want violence or not is because it plays to our human nature. The football arena is the new Colosseum, and the players are the new gladiators. One of the most powerful human instinct is the instinct of survival. Naturally, then, we are entertained when people are in peril because we are instinctively drawn to their fight for survival. This doesn’t make us evil; we don’t want to see people get hurt. We do, however, want to see people get hit.

(4) The problem is that there is no definitive solution to violence in sports. We want it and don’t want it at the same time. It seems our culture has decided to allow it in certain areas where we think it belongs, like mixed martial arts, but condemning it where we think it doesn’t, like football. Football fans worry about what’s going to happen to the sport in the next few years as reports of brain injuries continue to surface. Most likely, though, nothing will change. We will sit around our dinner table and argue that something needs to be done, then head to the Colosseum and cheer for every big hit.
 

Read this sentence from Paragraph 4:

We will sit around our dinner table and argue that something needs to be done, then head to the Colosseum and cheer for every big hit.

Why does the author use the phrase “dinner table” in this sentence?

A
To stress the idea that people only find violence a problem in sports when they are trying to eat a meal and don’t want that kind of imagery in their minds.
B
To point that people don’t actually share their true feelings at the “dinner table” because they are afraid of what their family will think.
C
To emphasize the idea that people are anti-violent sports when they are discussing the problem at home at the “dinner table,” but act differently when they are watching a sporting event.
D
To highlight the point that violence in sports has gotten so prevalent that it’s even invading our time with our family at home.
Question 9 Explanation: 
The author is using the “dinner table” as juxtaposition against “the Colosseum” to emphasize her point that we may be against violent sports in theory, but entertained by it in practice. None of the other answer choices really address this comparison.
Question 10
(1) Social networking sites and apps provide global connectivity and allow people to make instant connections with each other. As social networking becomes more ubiquitous in modern society, employers and admissions offices are starting to use these sites as a way to learn more about potential candidates and ultimately inform their decisions. This means that you could lose out on a chance to go to the college you want, or work at your dream job, just because of a comment you made on social media.

(2) In a recent study, only 30% of high school students were “extremely concerned” about college admission offices using their social networking information as part of their admission process (41% of students were “not at all concerned”). Because social networking platforms usually have a “private” setting that gives users a feeling of security, they think they have control over what information is accessible to employers and colleges. However, social networking platforms are supposed to be a place where people can express themselves, share their lives with friends and family, and meet new people. Once employers and colleges start snooping in on these platforms, people will be forced to cultivate an “acceptable” version of themselves. This requires censorship—self-censorship perhaps, but censorship nonetheless. Additionally, this kind of invasion could lead to discrimination, discrimination that is impossible to monitor.

(3) American’s have laws that protect them from discrimination by employers and colleges. A job interviewer, for example, can’t directly ask a candidate about their age, sexual preference, religious beliefs, or political ideals. These questions lead to discrimination. The average person’s social networking activity answers these questions, and leaves them vulnerable to discrimination.

(4) Even though people have some control over what information they publicize on social networking platforms, when employers and colleges pore through this information, they are invading the personal lives of their candidates. For many Americans, their work life is already invading their personal life way too much. With inflation climbing and wages remaining stagnant, modern citizens are less able to have a life outside of work. Now, employers want to invade what small social life people do have. It’s important for our own mental health that we can keep these parts of our life separate.

(5) Employers and colleges might say that they are doing it to weed out people with extreme behavior. An employer may not want to hire someone who has posted anti-Semitic hate speech all over the internet, for example. These extreme examples don’t justify monitoring everyone, though, much like trying to catch one terrorist doesn’t justify the government listening in on everyone’s phone calls. People deserve to have some privacy in their lives, and employers and colleges need to respect that.
 

Read this sentence from Paragraph 1:

As social networking becomes more ubiquitous in modern society, employers and admissions offices are starting to use these sites as a way to learn more about potential candidates and ultimately inform their decisions.

In the context of the sentence, what does “ubiquitous” most nearly mean?

A
Seemingly everywhere
B
Important
C
Necessary
D
Harmful
Question 10 Explanation: 
Answer choice (A) is correct because it most nearly reflects the definition of “ubiquitous.” The author is commenting that social networking is everywhere in modern society and only growing. He is not commenting on its importance (B) or necessity (C), nor is he suggestion that the growing presence of these platforms is harmful (D).
Question 11
(1) Social networking sites and apps provide global connectivity and allow people to make instant connections with each other. As social networking becomes more ubiquitous in modern society, employers and admissions offices are starting to use these sites as a way to learn more about potential candidates and ultimately inform their decisions. This means that you could lose out on a chance to go to the college you want, or work at your dream job, just because of a comment you made on social media.

(2) In a recent study, only 30% of high school students were “extremely concerned” about college admission offices using their social networking information as part of their admission process (41% of students were “not at all concerned”). Because social networking platforms usually have a “private” setting that gives users a feeling of security, they think they have control over what information is accessible to employers and colleges. However, social networking platforms are supposed to be a place where people can express themselves, share their lives with friends and family, and meet new people. Once employers and colleges start snooping in on these platforms, people will be forced to cultivate an “acceptable” version of themselves. This requires censorship—self-censorship perhaps, but censorship nonetheless. Additionally, this kind of invasion could lead to discrimination, discrimination that is impossible to monitor.

(3) American’s have laws that protect them from discrimination by employers and colleges. A job interviewer, for example, can’t directly ask a candidate about their age, sexual preference, religious beliefs, or political ideals. These questions lead to discrimination. The average person’s social networking activity answers these questions, and leaves them vulnerable to discrimination.

(4) Even though people have some control over what information they publicize on social networking platforms, when employers and colleges pore through this information, they are invading the personal lives of their candidates. For many Americans, their work life is already invading their personal life way too much. With inflation climbing and wages remaining stagnant, modern citizens are less able to have a life outside of work. Now, employers want to invade what small social life people do have. It’s important for our own mental health that we can keep these parts of our life separate.

(5) Employers and colleges might say that they are doing it to weed out people with extreme behavior. An employer may not want to hire someone who has posted anti-Semitic hate speech all over the internet, for example. These extreme examples don’t justify monitoring everyone, though, much like trying to catch one terrorist doesn’t justify the government listening in on everyone’s phone calls. People deserve to have some privacy in their lives, and employers and colleges need to respect that.
 

Part A

Which statement expresses a claim the author makes about the consequences of employers and colleges monitoring social media?

A
It’s healthy for people to have a social outlet like what is provided for them on social networking platforms.
B
People need to be more careful about what they post because employers and colleges are watching.
C
People can’t be expected to censor themselves all of the time because it would be too hard.
D
Any employer or college that uses social networking platforms to inform hiring/admitting decisions is discriminating against the candidates they are reviewing.
Question 11 Explanation: 
The author explicitly discusses how a personal life is healthy, and how social media platforms are a part of our social lives. Answer choice (A) most accurately expresses this idea. While the author may agree with answer choice (B) and (C), these ideas are not reflected in the article.
Question 12
(1) Social networking sites and apps provide global connectivity and allow people to make instant connections with each other. As social networking becomes more ubiquitous in modern society, employers and admissions offices are starting to use these sites as a way to learn more about potential candidates and ultimately inform their decisions. This means that you could lose out on a chance to go to the college you want, or work at your dream job, just because of a comment you made on social media.

(2) In a recent study, only 30% of high school students were “extremely concerned” about college admission offices using their social networking information as part of their admission process (41% of students were “not at all concerned”). Because social networking platforms usually have a “private” setting that gives users a feeling of security, they think they have control over what information is accessible to employers and colleges. However, social networking platforms are supposed to be a place where people can express themselves, share their lives with friends and family, and meet new people. Once employers and colleges start snooping in on these platforms, people will be forced to cultivate an “acceptable” version of themselves. This requires censorship—self-censorship perhaps, but censorship nonetheless. Additionally, this kind of invasion could lead to discrimination, discrimination that is impossible to monitor.

(3) American’s have laws that protect them from discrimination by employers and colleges. A job interviewer, for example, can’t directly ask a candidate about their age, sexual preference, religious beliefs, or political ideals. These questions lead to discrimination. The average person’s social networking activity answers these questions, and leaves them vulnerable to discrimination.

(4) Even though people have some control over what information they publicize on social networking platforms, when employers and colleges pore through this information, they are invading the personal lives of their candidates. For many Americans, their work life is already invading their personal life way too much. With inflation climbing and wages remaining stagnant, modern citizens are less able to have a life outside of work. Now, employers want to invade what small social life people do have. It’s important for our own mental health that we can keep these parts of our life separate.

(5) Employers and colleges might say that they are doing it to weed out people with extreme behavior. An employer may not want to hire someone who has posted anti-Semitic hate speech all over the internet, for example. These extreme examples don’t justify monitoring everyone, though, much like trying to catch one terrorist doesn’t justify the government listening in on everyone’s phone calls. People deserve to have some privacy in their lives, and employers and colleges need to respect that.
 

Part B

Which sentence from the article best supports the answer to Part A?

A
“American’s have laws that protect them from discrimination by employers and colleges.”
B
“This means that you could lose out on a chance to go to the college you want, or work at your dream job, just because of a comment you made on social media.”
C
“It’s important for our own mental health that we can keep these parts of our life separate.”
D
“An employer may not want to hire someone who has posted anti-Semitic hate speech all over the internet, for example.”
Question 12 Explanation: 
Answer choice (C) is the only option that addresses the importance of people separating their work lives from their social lives.
Question 13
(1) Social networking sites and apps provide global connectivity and allow people to make instant connections with each other. As social networking becomes more ubiquitous in modern society, employers and admissions offices are starting to use these sites as a way to learn more about potential candidates and ultimately inform their decisions. This means that you could lose out on a chance to go to the college you want, or work at your dream job, just because of a comment you made on social media.

(2) In a recent study, only 30% of high school students were “extremely concerned” about college admission offices using their social networking information as part of their admission process (41% of students were “not at all concerned”). Because social networking platforms usually have a “private” setting that gives users a feeling of security, they think they have control over what information is accessible to employers and colleges. However, social networking platforms are supposed to be a place where people can express themselves, share their lives with friends and family, and meet new people. Once employers and colleges start snooping in on these platforms, people will be forced to cultivate an “acceptable” version of themselves. This requires censorship—self-censorship perhaps, but censorship nonetheless. Additionally, this kind of invasion could lead to discrimination, discrimination that is impossible to monitor.

(3) American’s have laws that protect them from discrimination by employers and colleges. A job interviewer, for example, can’t directly ask a candidate about their age, sexual preference, religious beliefs, or political ideals. These questions lead to discrimination. The average person’s social networking activity answers these questions, and leaves them vulnerable to discrimination.

(4) Even though people have some control over what information they publicize on social networking platforms, when employers and colleges pore through this information, they are invading the personal lives of their candidates. For many Americans, their work life is already invading their personal life way too much. With inflation climbing and wages remaining stagnant, modern citizens are less able to have a life outside of work. Now, employers want to invade what small social life people do have. It’s important for our own mental health that we can keep these parts of our life separate.

(5) Employers and colleges might say that they are doing it to weed out people with extreme behavior. An employer may not want to hire someone who has posted anti-Semitic hate speech all over the internet, for example. These extreme examples don’t justify monitoring everyone, though, much like trying to catch one terrorist doesn’t justify the government listening in on everyone’s phone calls. People deserve to have some privacy in their lives, and employers and colleges need to respect that.
 

Part A

What is the central idea of the article?

A
The government needs to create laws that allow employers and colleges to use social networking platforms in their decision-making process, but regulate how they can use information gathered on these sites.
B
People’s privacy is important and it should be unlawful for employers and colleges to snoop into people’s private lives.
C
People should be more careful about what they post online because if they make something public, they should expect that everyone, including potential employers and colleges, will see it and judge them.
D
People should use social networking platforms as a way to cultivate a persona that will be acceptable to employers and colleges.
Question 13 Explanation: 
In the article, the author consistently stresses his concerns about employers and colleges using social media in their decision process. Answer choice (B) is correct because the author asserts that this practice should be ended completely, not regulated by the government (A), acknowledged by users (C), or manipulated by potential candidates.
Question 14
(1) Social networking sites and apps provide global connectivity and allow people to make instant connections with each other. As social networking becomes more ubiquitous in modern society, employers and admissions offices are starting to use these sites as a way to learn more about potential candidates and ultimately inform their decisions. This means that you could lose out on a chance to go to the college you want, or work at your dream job, just because of a comment you made on social media.

(2) In a recent study, only 30% of high school students were “extremely concerned” about college admission offices using their social networking information as part of their admission process (41% of students were “not at all concerned”). Because social networking platforms usually have a “private” setting that gives users a feeling of security, they think they have control over what information is accessible to employers and colleges. However, social networking platforms are supposed to be a place where people can express themselves, share their lives with friends and family, and meet new people. Once employers and colleges start snooping in on these platforms, people will be forced to cultivate an “acceptable” version of themselves. This requires censorship—self-censorship perhaps, but censorship nonetheless. Additionally, this kind of invasion could lead to discrimination, discrimination that is impossible to monitor.

(3) American’s have laws that protect them from discrimination by employers and colleges. A job interviewer, for example, can’t directly ask a candidate about their age, sexual preference, religious beliefs, or political ideals. These questions lead to discrimination. The average person’s social networking activity answers these questions, and leaves them vulnerable to discrimination.

(4) Even though people have some control over what information they publicize on social networking platforms, when employers and colleges pore through this information, they are invading the personal lives of their candidates. For many Americans, their work life is already invading their personal life way too much. With inflation climbing and wages remaining stagnant, modern citizens are less able to have a life outside of work. Now, employers want to invade what small social life people do have. It’s important for our own mental health that we can keep these parts of our life separate.

(5) Employers and colleges might say that they are doing it to weed out people with extreme behavior. An employer may not want to hire someone who has posted anti-Semitic hate speech all over the internet, for example. These extreme examples don’t justify monitoring everyone, though, much like trying to catch one terrorist doesn’t justify the government listening in on everyone’s phone calls. People deserve to have some privacy in their lives, and employers and colleges need to respect that.
 

Part B

Which sentence from the article best supports the correct answer from Part A?

A
“Social networking sites and apps provide global connectivity and allow people to make instant connections with each other.”
B
“A job interviewer, for example, can’t directly ask a candidate about their age, sexual preference, religious beliefs, or political ideals.”
C
“People deserve to have some privacy in their lives, and employers and colleges need to respect that.”
D
“In a recent study, only 30% of high school students were “extremely concerned” about college admission offices using their social networking information as part of their admission process (41% of students were “not at all concerned”).”
Question 14 Explanation: 
Option (C) is the only answer that directly supports the central idea of Part A.
Question 15
(1) Social networking sites and apps provide global connectivity and allow people to make instant connections with each other. As social networking becomes more ubiquitous in modern society, employers and admissions offices are starting to use these sites as a way to learn more about potential candidates and ultimately inform their decisions. This means that you could lose out on a chance to go to the college you want, or work at your dream job, just because of a comment you made on social media.

(2) In a recent study, only 30% of high school students were “extremely concerned” about college admission offices using their social networking information as part of their admission process (41% of students were “not at all concerned”). Because social networking platforms usually have a “private” setting that gives users a feeling of security, they think they have control over what information is accessible to employers and colleges. However, social networking platforms are supposed to be a place where people can express themselves, share their lives with friends and family, and meet new people. Once employers and colleges start snooping in on these platforms, people will be forced to cultivate an “acceptable” version of themselves. This requires censorship—self-censorship perhaps, but censorship nonetheless. Additionally, this kind of invasion could lead to discrimination, discrimination that is impossible to monitor.

(3) American’s have laws that protect them from discrimination by employers and colleges. A job interviewer, for example, can’t directly ask a candidate about their age, sexual preference, religious beliefs, or political ideals. These questions lead to discrimination. The average person’s social networking activity answers these questions, and leaves them vulnerable to discrimination.

(4) Even though people have some control over what information they publicize on social networking platforms, when employers and colleges pore through this information, they are invading the personal lives of their candidates. For many Americans, their work life is already invading their personal life way too much. With inflation climbing and wages remaining stagnant, modern citizens are less able to have a life outside of work. Now, employers want to invade what small social life people do have. It’s important for our own mental health that we can keep these parts of our life separate.

(5) Employers and colleges might say that they are doing it to weed out people with extreme behavior. An employer may not want to hire someone who has posted anti-Semitic hate speech all over the internet, for example. These extreme examples don’t justify monitoring everyone, though, much like trying to catch one terrorist doesn’t justify the government listening in on everyone’s phone calls. People deserve to have some privacy in their lives, and employers and colleges need to respect that.
 

Which of the following words in the article is specifically chosen to influence readers to agree with the author?

A
“snooping,” Paragraph 2
B
“connectivity,” Paragraph 1
C
“publicize,” Paragraph 4
D
“weed,” Paragraph 5
Question 15 Explanation: 
Answer choice (A) is the correct answer because the author specifically uses the word “snooping” to make employers and colleges look bad. “Snooping” has the connotation of someone unfairly watching you, as if they are crouching in the bushes outside of your door with binoculars. None of the other words have a negative connotation that specifically influences readers to agree with the author’s point.
Question 16
(1) Social networking sites and apps provide global connectivity and allow people to make instant connections with each other. As social networking becomes more ubiquitous in modern society, employers and admissions offices are starting to use these sites as a way to learn more about potential candidates and ultimately inform their decisions. This means that you could lose out on a chance to go to the college you want, or work at your dream job, just because of a comment you made on social media.

(2) In a recent study, only 30% of high school students were “extremely concerned” about college admission offices using their social networking information as part of their admission process (41% of students were “not at all concerned”). Because social networking platforms usually have a “private” setting that gives users a feeling of security, they think they have control over what information is accessible to employers and colleges. However, social networking platforms are supposed to be a place where people can express themselves, share their lives with friends and family, and meet new people. Once employers and colleges start snooping in on these platforms, people will be forced to cultivate an “acceptable” version of themselves. This requires censorship—self-censorship perhaps, but censorship nonetheless. Additionally, this kind of invasion could lead to discrimination, discrimination that is impossible to monitor.

(3) American’s have laws that protect them from discrimination by employers and colleges. A job interviewer, for example, can’t directly ask a candidate about their age, sexual preference, religious beliefs, or political ideals. These questions lead to discrimination. The average person’s social networking activity answers these questions, and leaves them vulnerable to discrimination.

(4) Even though people have some control over what information they publicize on social networking platforms, when employers and colleges pore through this information, they are invading the personal lives of their candidates. For many Americans, their work life is already invading their personal life way too much. With inflation climbing and wages remaining stagnant, modern citizens are less able to have a life outside of work. Now, employers want to invade what small social life people do have. It’s important for our own mental health that we can keep these parts of our life separate.

(5) Employers and colleges might say that they are doing it to weed out people with extreme behavior. An employer may not want to hire someone who has posted anti-Semitic hate speech all over the internet, for example. These extreme examples don’t justify monitoring everyone, though, much like trying to catch one terrorist doesn’t justify the government listening in on everyone’s phone calls. People deserve to have some privacy in their lives, and employers and colleges need to respect that.
 

Read the sentence from paragraph 2.

In a recent study, only 30% of high school students were “extremely concerned” about college admission offices using their social networking information as part of their admission process (41% of students were “not at all concerned”).

What is the problem (if any) with the statistic provided in the sentence above?

A
When you add the percentages together, the math doesn’t add up.
B
The data collected doesn’t consider important ideas that would affect any conclusions you could make from it.
C
The indicators “extremely concerned” and “not at all concerned” are not defined, quantifiable data points.
D
The author doesn’t indicate where the study is from or who conducted it, so the audience doesn’t know if the data presented reflects a defendable statistic.
Question 16 Explanation: 
Answer choice (D) is correct because it’s important for writers to provide a source whenever they cite data. This will help the reader better understand whether the data is valid or not. Answer choice (C) might be tempting, but the author indicates that this information is from a survey, which means that even if the indicators are subjective, they become quantifiable data points as soon as subjects of the survey begin choosing them.
Question 17
(1) Social networking sites and apps provide global connectivity and allow people to make instant connections with each other. As social networking becomes more ubiquitous in modern society, employers and admissions offices are starting to use these sites as a way to learn more about potential candidates and ultimately inform their decisions. This means that you could lose out on a chance to go to the college you want, or work at your dream job, just because of a comment you made on social media.

(2) In a recent study, only 30% of high school students were “extremely concerned” about college admission offices using their social networking information as part of their admission process (41% of students were “not at all concerned”). Because social networking platforms usually have a “private” setting that gives users a feeling of security, they think they have control over what information is accessible to employers and colleges. However, social networking platforms are supposed to be a place where people can express themselves, share their lives with friends and family, and meet new people. Once employers and colleges start snooping in on these platforms, people will be forced to cultivate an “acceptable” version of themselves. This requires censorship—self-censorship perhaps, but censorship nonetheless. Additionally, this kind of invasion could lead to discrimination, discrimination that is impossible to monitor.

(3) American’s have laws that protect them from discrimination by employers and colleges. A job interviewer, for example, can’t directly ask a candidate about their age, sexual preference, religious beliefs, or political ideals. These questions lead to discrimination. The average person’s social networking activity answers these questions, and leaves them vulnerable to discrimination.

(4) Even though people have some control over what information they publicize on social networking platforms, when employers and colleges pore through this information, they are invading the personal lives of their candidates. For many Americans, their work life is already invading their personal life way too much. With inflation climbing and wages remaining stagnant, modern citizens are less able to have a life outside of work. Now, employers want to invade what small social life people do have. It’s important for our own mental health that we can keep these parts of our life separate.

(5) Employers and colleges might say that they are doing it to weed out people with extreme behavior. An employer may not want to hire someone who has posted anti-Semitic hate speech all over the internet, for example. These extreme examples don’t justify monitoring everyone, though, much like trying to catch one terrorist doesn’t justify the government listening in on everyone’s phone calls. People deserve to have some privacy in their lives, and employers and colleges need to respect that.
 

Read the sentence from paragraph 5.

These extreme examples don’t justify monitoring everyone, though, much like trying to catch one terrorist doesn’t justify the government listening in on everyone’s phone calls.

Which of the following best explains how this comparison furthers the author’s main idea?

A
The author creates an image in the reader’s mind of employers and colleges as terrorists in order to scare them into agreeing with him.
B
The author uses a relatable example of what most people would consider an invasion of privacy, unwarranted phone taps, to highlight the practices of employers and colleges as unethical.
C
The author uses this comparison to segue into his assertion that the practices of employers and colleges needs to be regulated as much as law enforcement is when they want to tap someone’s phone.
D
The author wants to specifically target victims of unlawful investigations in this comparison because he believes they are an influential part of his audience.
Question 17 Explanation: 
The author is using a comparison to connect this problem to a problem of which his audience would already be aware. Answer choice (B) is the only option that clearly explains this. Answer choice (C) might be tempting, but the article doesn’t discuss regulating the practice at hand, but rather discusses ending it entirely.
Question 18
(1) Social networking sites and apps provide global connectivity and allow people to make instant connections with each other. As social networking becomes more ubiquitous in modern society, employers and admissions offices are starting to use these sites as a way to learn more about potential candidates and ultimately inform their decisions. This means that you could lose out on a chance to go to the college you want, or work at your dream job, just because of a comment you made on social media.

(2) In a recent study, only 30% of high school students were “extremely concerned” about college admission offices using their social networking information as part of their admission process (41% of students were “not at all concerned”). Because social networking platforms usually have a “private” setting that gives users a feeling of security, they think they have control over what information is accessible to employers and colleges. However, social networking platforms are supposed to be a place where people can express themselves, share their lives with friends and family, and meet new people. Once employers and colleges start snooping in on these platforms, people will be forced to cultivate an “acceptable” version of themselves. This requires censorship—self-censorship perhaps, but censorship nonetheless. Additionally, this kind of invasion could lead to discrimination, discrimination that is impossible to monitor.

(3) American’s have laws that protect them from discrimination by employers and colleges. A job interviewer, for example, can’t directly ask a candidate about their age, sexual preference, religious beliefs, or political ideals. These questions lead to discrimination. The average person’s social networking activity answers these questions, and leaves them vulnerable to discrimination.

(4) Even though people have some control over what information they publicize on social networking platforms, when employers and colleges pore through this information, they are invading the personal lives of their candidates. For many Americans, their work life is already invading their personal life way too much. With inflation climbing and wages remaining stagnant, modern citizens are less able to have a life outside of work. Now, employers want to invade what small social life people do have. It’s important for our own mental health that we can keep these parts of our life separate.

(5) Employers and colleges might say that they are doing it to weed out people with extreme behavior. An employer may not want to hire someone who has posted anti-Semitic hate speech all over the internet, for example. These extreme examples don’t justify monitoring everyone, though, much like trying to catch one terrorist doesn’t justify the government listening in on everyone’s phone calls. People deserve to have some privacy in their lives, and employers and colleges need to respect that.
 

How does Paragraph 3 contribute to the development of ideas in the article?

A
The author uses an analogy that he will continue to refer to throughout the rest of the article.
B
The author presents a counter-argument and refutes the opinions of people who would argue against him.
C
The author provides an ethical appeal as he explains why he is an expert on the subject of the article.
D
The author provides his first specific supporting ideas in defense of his overall purpose.
Question 18 Explanation: 
Up until Paragraph 3, the author has expressed his disapproval of the practice of employers and colleges using social networking with vague generalities. In this paragraph, he introduces the potential discriminatory implications of this practice.
Question 19
(1) Social networking sites and apps provide global connectivity and allow people to make instant connections with each other. As social networking becomes more ubiquitous in modern society, employers and admissions offices are starting to use these sites as a way to learn more about potential candidates and ultimately inform their decisions. This means that you could lose out on a chance to go to the college you want, or work at your dream job, just because of a comment you made on social media.

(2) In a recent study, only 30% of high school students were “extremely concerned” about college admission offices using their social networking information as part of their admission process (41% of students were “not at all concerned”). Because social networking platforms usually have a “private” setting that gives users a feeling of security, they think they have control over what information is accessible to employers and colleges. However, social networking platforms are supposed to be a place where people can express themselves, share their lives with friends and family, and meet new people. Once employers and colleges start snooping in on these platforms, people will be forced to cultivate an “acceptable” version of themselves. This requires censorship—self-censorship perhaps, but censorship nonetheless. Additionally, this kind of invasion could lead to discrimination, discrimination that is impossible to monitor.

(3) American’s have laws that protect them from discrimination by employers and colleges. A job interviewer, for example, can’t directly ask a candidate about their age, sexual preference, religious beliefs, or political ideals. These questions lead to discrimination. The average person’s social networking activity answers these questions, and leaves them vulnerable to discrimination.

(4) Even though people have some control over what information they publicize on social networking platforms, when employers and colleges pore through this information, they are invading the personal lives of their candidates. For many Americans, their work life is already invading their personal life way too much. With inflation climbing and wages remaining stagnant, modern citizens are less able to have a life outside of work. Now, employers want to invade what small social life people do have. It’s important for our own mental health that we can keep these parts of our life separate.

(5) Employers and colleges might say that they are doing it to weed out people with extreme behavior. An employer may not want to hire someone who has posted anti-Semitic hate speech all over the internet, for example. These extreme examples don’t justify monitoring everyone, though, much like trying to catch one terrorist doesn’t justify the government listening in on everyone’s phone calls. People deserve to have some privacy in their lives, and employers and colleges need to respect that.
 

Which of the following is an inference that readers can make from what they read in paragraph 3?

A
When an employer or college researches a candidate on social networking sites, they are discriminating against that candidate.
B
People should delete information from their social networking profiles that could lead to discrimination, like their gender, orientation, and religious or political affiliations.
C
There have been several lawsuits already in which candidates have sued companies over discrimination because these companies used the candidates’ social networking activity as a reason to disqualify them.
D
Employers and colleges that use social networking to research their candidates could be vulnerable to lawsuits for discrimination.
Question 19 Explanation: 
Answer choice (D) is correct because the author merely points out how this practice could potentially put a company in a vulnerable position, rather than asserting that discrimination is currently happening, as stated/suggested in answer choices (A) and (C).
Question 20
The Modern Horror Movie

(1) The American Film Industry, like any business market, has a history of adapting itself to meet the demands of its customers. When viewers respond to a film trend, movies studios do their best to capitalize on that trend. This can sometimes lead to disastrous losses, but it has also helped to shape American films throughout the history of the industry. Summer blockbusters, for example, have become an annual tradition. Up until the last few years, studios almost exclusively released their big-budget films during the summer (ticket sales have recently proven that these big budget films can flourish during other seasons as well).

(2) Summer wasn’t always the time for big budget movies. In fact, summer used to be the slow season for movie studios. People were out at the beach all day and didn’t want to spend their evenings inside of a movie theater. Many critics trace the modern summer blockbuster back to Jaws in 1975, when director Steven Spielberg and Universal deliberately delayed the opening of their film to coincide with summer. Since Jaws is a film about a shark terrorizing a sleepy beach town, Spielberg and Universal figured it would have the most impact during the season when people frequented the beach. They were right. After Jaws, more movies began dipping their toes into the summer market. Spielberg’s friend, George Lucas, released each of the films in his original Star Wars trilogy during the summer. By the 90’s big-budget summer blockbusters were an annual tradition.

(3) In the last 20 years, however, we have seen a trend in films that is almost the antithesis of the big-budget summer blockbusters. Studios have proven through the last 40 years that big budget movies can pay big dividends. In the last 20 years, though, some movie studios are cashing in on the fact that small budget movies can have even better returns. A contemporary big-budget blockbuster can cost around $250 million to make, and another $250 million to properly market. If that film makes $1 billion, then the movie studio has made $500 million, or 100% profit. That is an amazing return on the studio’s investment. However, if that same studio made a smaller film for $5 million, and spent another $5 million on marketing, they could reach the same percentage of return on their investment if the movie makes $10 million. The justification behind big budget movies is that betting big potentially gives the studio the biggest payoff. The downfall, however, is that the studios are shelling out a lot of money up front; if the movie doesn’t do great, it’s a failure.

(4) If you do some research on the highest grossing movies of all time, the list is exclusively big-budget movies. The top five movies on the list, Avatar, Titanic, Star Wars: The Force Awakens, Avengers: Infinity War, and Jurassic World cost over $1 billion to be made (not counting the money spent on marketing). However, if you do some research on the most profitable movies of all time, none of the films on the list cost even $1 million to make.

(5) The studios that have embraced this low-budget, high return trend the most are studios who produce horror movies. Since the release of The Blair Witch Project in 1999, a film that cost $600,000 to make, and grossed over $200 million, studios have realized that horror films can make them a lot of money with very little risk. Some criticize this trend because studios can continue to roll out sequels to movies like Paranormal Activity (the most profitable movie of all time by most reports) and The Purge even after they are no longer good. They cost very little to make and their brand recognition means they will likely make a decent run at the box office. However, the low financial risk of these films also gives studios the opportunity to give new filmmakers more freedom. A studio producing a big budget film would never hand over the reins to a newcomer without overseeing every step of the production carefully. Because of the low risk, Jordan Peele, a first-time director known for a comedy sketch show, had the freedom to do what he wants with his first movie. The result of this first-time, unproven director getting directorial freedom is Get Out, one of the most critically-acclaimed movies of 2017. This trend will invariably lead to more lazy horror sequels, but it could also lead to the de-stigmatization of the horror genre. Jordan Peele received an Oscar for Original Screenplay for Get Out; is a Best Picture-winning horror film far off?
 

Read this sentence from the passage:

A studio producing a big budget film would never hand over the reins to a newcomer without overseeing every step of the production carefully.

Which of these phrases best explains the meaning of the phrase “hand over the reins”?

A
Give complete control
B
Ride like a horse
C
Implicitly distrust
D
Take a risk
Question 20 Explanation: 
The correct answer is answer choice (A). The sentence is discussing how studios would not trust a newcomer completely with a big budget film. In this case, “hand over the reins” means “to give complete control.” Answer choice (D) is tempting because studios would be taking a risk if they were to do this, but the phrase itself doesn’t convey that sense of risk.
Question 21
The Modern Horror Movie

(1) The American Film Industry, like any business market, has a history of adapting itself to meet the demands of its customers. When viewers respond to a film trend, movies studios do their best to capitalize on that trend. This can sometimes lead to disastrous losses, but it has also helped to shape American films throughout the history of the industry. Summer blockbusters, for example, have become an annual tradition. Up until the last few years, studios almost exclusively released their big-budget films during the summer (ticket sales have recently proven that these big budget films can flourish during other seasons as well).

(2) Summer wasn’t always the time for big budget movies. In fact, summer used to be the slow season for movie studios. People were out at the beach all day and didn’t want to spend their evenings inside of a movie theater. Many critics trace the modern summer blockbuster back to Jaws in 1975, when director Steven Spielberg and Universal deliberately delayed the opening of their film to coincide with summer. Since Jaws is a film about a shark terrorizing a sleepy beach town, Spielberg and Universal figured it would have the most impact during the season when people frequented the beach. They were right. After Jaws, more movies began dipping their toes into the summer market. Spielberg’s friend, George Lucas, released each of the films in his original Star Wars trilogy during the summer. By the 90’s big-budget summer blockbusters were an annual tradition.

(3) In the last 20 years, however, we have seen a trend in films that is almost the antithesis of the big-budget summer blockbusters. Studios have proven through the last 40 years that big budget movies can pay big dividends. In the last 20 years, though, some movie studios are cashing in on the fact that small budget movies can have even better returns. A contemporary big-budget blockbuster can cost around $250 million to make, and another $250 million to properly market. If that film makes $1 billion, then the movie studio has made $500 million, or 100% profit. That is an amazing return on the studio’s investment. However, if that same studio made a smaller film for $5 million, and spent another $5 million on marketing, they could reach the same percentage of return on their investment if the movie makes $10 million. The justification behind big budget movies is that betting big potentially gives the studio the biggest payoff. The downfall, however, is that the studios are shelling out a lot of money up front; if the movie doesn’t do great, it’s a failure.

(4) If you do some research on the highest grossing movies of all time, the list is exclusively big-budget movies. The top five movies on the list, Avatar, Titanic, Star Wars: The Force Awakens, Avengers: Infinity War, and Jurassic World cost over $1 billion to be made (not counting the money spent on marketing). However, if you do some research on the most profitable movies of all time, none of the films on the list cost even $1 million to make.

(5) The studios that have embraced this low-budget, high return trend the most are studios who produce horror movies. Since the release of The Blair Witch Project in 1999, a film that cost $600,000 to make, and grossed over $200 million, studios have realized that horror films can make them a lot of money with very little risk. Some criticize this trend because studios can continue to roll out sequels to movies like Paranormal Activity (the most profitable movie of all time by most reports) and The Purge even after they are no longer good. They cost very little to make and their brand recognition means they will likely make a decent run at the box office. However, the low financial risk of these films also gives studios the opportunity to give new filmmakers more freedom. A studio producing a big budget film would never hand over the reins to a newcomer without overseeing every step of the production carefully. Because of the low risk, Jordan Peele, a first-time director known for a comedy sketch show, had the freedom to do what he wants with his first movie. The result of this first-time, unproven director getting directorial freedom is Get Out, one of the most critically-acclaimed movies of 2017. This trend will invariably lead to more lazy horror sequels, but it could also lead to the de-stigmatization of the horror genre. Jordan Peele received an Oscar for Original Screenplay for Get Out; is a Best Picture-winning horror film far off?
 

How does paragraph 2 contribute to the development of ideas in the passage?

A
It acts as a counter-argument to the major idea the writer discusses.
B
It presents specific information about how much money certain movies have made.
C
It discusses the steps that led to the current surge of low-budget horror films.
D
It discusses the history of a well-known movie trend.
Question 21 Explanation: 
The correct answer is answer choice (D). This paragraph discusses how Jaws is widely considered the first big summer blockbuster, and how that movie sparked a trend that still affects movies today. This is not a counter-argument (A) because in later paragraphs, the writer discusses how low-budget horror films are a more recent trend, but that their evolution is a lot like the evolution of summer blockbusters. Answer choices (B) and (C) are accurate descriptions of paragraphs 3 and 5, respectively, rather than paragraph 2.
Question 22
The Modern Horror Movie

(1) The American Film Industry, like any business market, has a history of adapting itself to meet the demands of its customers. When viewers respond to a film trend, movies studios do their best to capitalize on that trend. This can sometimes lead to disastrous losses, but it has also helped to shape American films throughout the history of the industry. Summer blockbusters, for example, have become an annual tradition. Up until the last few years, studios almost exclusively released their big-budget films during the summer (ticket sales have recently proven that these big budget films can flourish during other seasons as well).

(2) Summer wasn’t always the time for big budget movies. In fact, summer used to be the slow season for movie studios. People were out at the beach all day and didn’t want to spend their evenings inside of a movie theater. Many critics trace the modern summer blockbuster back to Jaws in 1975, when director Steven Spielberg and Universal deliberately delayed the opening of their film to coincide with summer. Since Jaws is a film about a shark terrorizing a sleepy beach town, Spielberg and Universal figured it would have the most impact during the season when people frequented the beach. They were right. After Jaws, more movies began dipping their toes into the summer market. Spielberg’s friend, George Lucas, released each of the films in his original Star Wars trilogy during the summer. By the 90’s big-budget summer blockbusters were an annual tradition.

(3) In the last 20 years, however, we have seen a trend in films that is almost the antithesis of the big-budget summer blockbusters. Studios have proven through the last 40 years that big budget movies can pay big dividends. In the last 20 years, though, some movie studios are cashing in on the fact that small budget movies can have even better returns. A contemporary big-budget blockbuster can cost around $250 million to make, and another $250 million to properly market. If that film makes $1 billion, then the movie studio has made $500 million, or 100% profit. That is an amazing return on the studio’s investment. However, if that same studio made a smaller film for $5 million, and spent another $5 million on marketing, they could reach the same percentage of return on their investment if the movie makes $10 million. The justification behind big budget movies is that betting big potentially gives the studio the biggest payoff. The downfall, however, is that the studios are shelling out a lot of money up front; if the movie doesn’t do great, it’s a failure.

(4) If you do some research on the highest grossing movies of all time, the list is exclusively big-budget movies. The top five movies on the list, Avatar, Titanic, Star Wars: The Force Awakens, Avengers: Infinity War, and Jurassic World cost over $1 billion to be made (not counting the money spent on marketing). However, if you do some research on the most profitable movies of all time, none of the films on the list cost even $1 million to make.

(5) The studios that have embraced this low-budget, high return trend the most are studios who produce horror movies. Since the release of The Blair Witch Project in 1999, a film that cost $600,000 to make, and grossed over $200 million, studios have realized that horror films can make them a lot of money with very little risk. Some criticize this trend because studios can continue to roll out sequels to movies like Paranormal Activity (the most profitable movie of all time by most reports) and The Purge even after they are no longer good. They cost very little to make and their brand recognition means they will likely make a decent run at the box office. However, the low financial risk of these films also gives studios the opportunity to give new filmmakers more freedom. A studio producing a big budget film would never hand over the reins to a newcomer without overseeing every step of the production carefully. Because of the low risk, Jordan Peele, a first-time director known for a comedy sketch show, had the freedom to do what he wants with his first movie. The result of this first-time, unproven director getting directorial freedom is Get Out, one of the most critically-acclaimed movies of 2017. This trend will invariably lead to more lazy horror sequels, but it could also lead to the de-stigmatization of the horror genre. Jordan Peele received an Oscar for Original Screenplay for Get Out; is a Best Picture-winning horror film far off?
 

How does the story of Jaws in paragraph 2 connect to the story of The Blair Witch Project in paragraph 5?

A
Both movies are horror films from the 1970’s.
B
Both films were big budget summer movies.
C
Both stories are about a film that proved the viability of a new film trend.
D
Both films were the result of movies studios capitalizing on established trends.
Question 22 Explanation: 
The correct answer is answer choice (C). The writer connects Jaws, generally accepted as the first big budget summer blockbuster, to The Blair Witch Project, which sparked the modern low-budget horror films we see today. Each of them is arguably the first film in a popular trend. Answer choices (A) and (B) are directly refuted by the passage, and answer choice (D) is incorrect because neither movie was capitalized on an established trend.
Question 23
The Modern Horror Movie

(1) The American Film Industry, like any business market, has a history of adapting itself to meet the demands of its customers. When viewers respond to a film trend, movies studios do their best to capitalize on that trend. This can sometimes lead to disastrous losses, but it has also helped to shape American films throughout the history of the industry. Summer blockbusters, for example, have become an annual tradition. Up until the last few years, studios almost exclusively released their big-budget films during the summer (ticket sales have recently proven that these big budget films can flourish during other seasons as well).

(2) Summer wasn’t always the time for big budget movies. In fact, summer used to be the slow season for movie studios. People were out at the beach all day and didn’t want to spend their evenings inside of a movie theater. Many critics trace the modern summer blockbuster back to Jaws in 1975, when director Steven Spielberg and Universal deliberately delayed the opening of their film to coincide with summer. Since Jaws is a film about a shark terrorizing a sleepy beach town, Spielberg and Universal figured it would have the most impact during the season when people frequented the beach. They were right. After Jaws, more movies began dipping their toes into the summer market. Spielberg’s friend, George Lucas, released each of the films in his original Star Wars trilogy during the summer. By the 90’s big-budget summer blockbusters were an annual tradition.

(3) In the last 20 years, however, we have seen a trend in films that is almost the antithesis of the big-budget summer blockbusters. Studios have proven through the last 40 years that big budget movies can pay big dividends. In the last 20 years, though, some movie studios are cashing in on the fact that small budget movies can have even better returns. A contemporary big-budget blockbuster can cost around $250 million to make, and another $250 million to properly market. If that film makes $1 billion, then the movie studio has made $500 million, or 100% profit. That is an amazing return on the studio’s investment. However, if that same studio made a smaller film for $5 million, and spent another $5 million on marketing, they could reach the same percentage of return on their investment if the movie makes $10 million. The justification behind big budget movies is that betting big potentially gives the studio the biggest payoff. The downfall, however, is that the studios are shelling out a lot of money up front; if the movie doesn’t do great, it’s a failure.

(4) If you do some research on the highest grossing movies of all time, the list is exclusively big-budget movies. The top five movies on the list, Avatar, Titanic, Star Wars: The Force Awakens, Avengers: Infinity War, and Jurassic World cost over $1 billion to be made (not counting the money spent on marketing). However, if you do some research on the most profitable movies of all time, none of the films on the list cost even $1 million to make.

(5) The studios that have embraced this low-budget, high return trend the most are studios who produce horror movies. Since the release of The Blair Witch Project in 1999, a film that cost $600,000 to make, and grossed over $200 million, studios have realized that horror films can make them a lot of money with very little risk. Some criticize this trend because studios can continue to roll out sequels to movies like Paranormal Activity (the most profitable movie of all time by most reports) and The Purge even after they are no longer good. They cost very little to make and their brand recognition means they will likely make a decent run at the box office. However, the low financial risk of these films also gives studios the opportunity to give new filmmakers more freedom. A studio producing a big budget film would never hand over the reins to a newcomer without overseeing every step of the production carefully. Because of the low risk, Jordan Peele, a first-time director known for a comedy sketch show, had the freedom to do what he wants with his first movie. The result of this first-time, unproven director getting directorial freedom is Get Out, one of the most critically-acclaimed movies of 2017. This trend will invariably lead to more lazy horror sequels, but it could also lead to the de-stigmatization of the horror genre. Jordan Peele received an Oscar for Original Screenplay for Get Out; is a Best Picture-winning horror film far off?
 

Which sentence in the passage best expresses the writer’s attitude towards the current trend of low-budget horror films?

A
The relatively low risk of these films always for new filmmakers to make their mark.
B
Low-budget horror films make more money at the box office than big budget blockbusters.
C
Audiences need to stop going to these movies so that studios will stop making them.
D
These films cannot be taken seriously.
Question 23 Explanation: 
The correct answer is answer choice (A). The writer does mention that some of these films are terrible, but he concludes the passage by discussing a specific example of how this trend made a great movie possible. The closing sentence also wonders whether this trend is going to change the way that critics receive horror movies. Answer choices (C) and (D) are inaccurate because the writer doesn’t condemn these films. Answer choice (B) might be tempting, but the article explores how low-budget movies can have a higher percentage return than big budget movies; it doesn’t suggest that low-budget movies make more money.
Question 24
The Modern Horror Movie

(1) The American Film Industry, like any business market, has a history of adapting itself to meet the demands of its customers. When viewers respond to a film trend, movies studios do their best to capitalize on that trend. This can sometimes lead to disastrous losses, but it has also helped to shape American films throughout the history of the industry. Summer blockbusters, for example, have become an annual tradition. Up until the last few years, studios almost exclusively released their big-budget films during the summer (ticket sales have recently proven that these big budget films can flourish during other seasons as well).

(2) Summer wasn’t always the time for big budget movies. In fact, summer used to be the slow season for movie studios. People were out at the beach all day and didn’t want to spend their evenings inside of a movie theater. Many critics trace the modern summer blockbuster back to Jaws in 1975, when director Steven Spielberg and Universal deliberately delayed the opening of their film to coincide with summer. Since Jaws is a film about a shark terrorizing a sleepy beach town, Spielberg and Universal figured it would have the most impact during the season when people frequented the beach. They were right. After Jaws, more movies began dipping their toes into the summer market. Spielberg’s friend, George Lucas, released each of the films in his original Star Wars trilogy during the summer. By the 90’s big-budget summer blockbusters were an annual tradition.

(3) In the last 20 years, however, we have seen a trend in films that is almost the antithesis of the big-budget summer blockbusters. Studios have proven through the last 40 years that big budget movies can pay big dividends. In the last 20 years, though, some movie studios are cashing in on the fact that small budget movies can have even better returns. A contemporary big-budget blockbuster can cost around $250 million to make, and another $250 million to properly market. If that film makes $1 billion, then the movie studio has made $500 million, or 100% profit. That is an amazing return on the studio’s investment. However, if that same studio made a smaller film for $5 million, and spent another $5 million on marketing, they could reach the same percentage of return on their investment if the movie makes $10 million. The justification behind big budget movies is that betting big potentially gives the studio the biggest payoff. The downfall, however, is that the studios are shelling out a lot of money up front; if the movie doesn’t do great, it’s a failure.

(4) If you do some research on the highest grossing movies of all time, the list is exclusively big-budget movies. The top five movies on the list, Avatar, Titanic, Star Wars: The Force Awakens, Avengers: Infinity War, and Jurassic World cost over $1 billion to be made (not counting the money spent on marketing). However, if you do some research on the most profitable movies of all time, none of the films on the list cost even $1 million to make.

(5) The studios that have embraced this low-budget, high return trend the most are studios who produce horror movies. Since the release of The Blair Witch Project in 1999, a film that cost $600,000 to make, and grossed over $200 million, studios have realized that horror films can make them a lot of money with very little risk. Some criticize this trend because studios can continue to roll out sequels to movies like Paranormal Activity (the most profitable movie of all time by most reports) and The Purge even after they are no longer good. They cost very little to make and their brand recognition means they will likely make a decent run at the box office. However, the low financial risk of these films also gives studios the opportunity to give new filmmakers more freedom. A studio producing a big budget film would never hand over the reins to a newcomer without overseeing every step of the production carefully. Because of the low risk, Jordan Peele, a first-time director known for a comedy sketch show, had the freedom to do what he wants with his first movie. The result of this first-time, unproven director getting directorial freedom is Get Out, one of the most critically-acclaimed movies of 2017. This trend will invariably lead to more lazy horror sequels, but it could also lead to the de-stigmatization of the horror genre. Jordan Peele received an Oscar for Original Screenplay for Get Out; is a Best Picture-winning horror film far off?
 

Part A

Which statement expresses a claim the author makes about big budget summer blockbusters in the passage?

A
They are not nearly as good as low-budget films.
B
Studios waste way too much money on big budget films.
C
Big budget movies always make more money than low-budget movies.
D
The risk of a big budget movie might be high, but the reward can be tremendous.
Question 24 Explanation: 
The correct answer is answer choice (D). The high risk and high reward of big budget films is a major discussion point for the writer. There is no evidence of answer choices (A) and (B) in the passage. Answer choice (C) is tempting because the movies that have made the most money are all big budget movies. However, not every big budget movie makes a lot of money. The word “always” makes this answer incorrect.
Question 25
The Modern Horror Movie

(1) The American Film Industry, like any business market, has a history of adapting itself to meet the demands of its customers. When viewers respond to a film trend, movies studios do their best to capitalize on that trend. This can sometimes lead to disastrous losses, but it has also helped to shape American films throughout the history of the industry. Summer blockbusters, for example, have become an annual tradition. Up until the last few years, studios almost exclusively released their big-budget films during the summer (ticket sales have recently proven that these big budget films can flourish during other seasons as well).

(2) Summer wasn’t always the time for big budget movies. In fact, summer used to be the slow season for movie studios. People were out at the beach all day and didn’t want to spend their evenings inside of a movie theater. Many critics trace the modern summer blockbuster back to Jaws in 1975, when director Steven Spielberg and Universal deliberately delayed the opening of their film to coincide with summer. Since Jaws is a film about a shark terrorizing a sleepy beach town, Spielberg and Universal figured it would have the most impact during the season when people frequented the beach. They were right. After Jaws, more movies began dipping their toes into the summer market. Spielberg’s friend, George Lucas, released each of the films in his original Star Wars trilogy during the summer. By the 90’s big-budget summer blockbusters were an annual tradition.

(3) In the last 20 years, however, we have seen a trend in films that is almost the antithesis of the big-budget summer blockbusters. Studios have proven through the last 40 years that big budget movies can pay big dividends. In the last 20 years, though, some movie studios are cashing in on the fact that small budget movies can have even better returns. A contemporary big-budget blockbuster can cost around $250 million to make, and another $250 million to properly market. If that film makes $1 billion, then the movie studio has made $500 million, or 100% profit. That is an amazing return on the studio’s investment. However, if that same studio made a smaller film for $5 million, and spent another $5 million on marketing, they could reach the same percentage of return on their investment if the movie makes $10 million. The justification behind big budget movies is that betting big potentially gives the studio the biggest payoff. The downfall, however, is that the studios are shelling out a lot of money up front; if the movie doesn’t do great, it’s a failure.

(4) If you do some research on the highest grossing movies of all time, the list is exclusively big-budget movies. The top five movies on the list, Avatar, Titanic, Star Wars: The Force Awakens, Avengers: Infinity War, and Jurassic World cost over $1 billion to be made (not counting the money spent on marketing). However, if you do some research on the most profitable movies of all time, none of the films on the list cost even $1 million to make.

(5) The studios that have embraced this low-budget, high return trend the most are studios who produce horror movies. Since the release of The Blair Witch Project in 1999, a film that cost $600,000 to make, and grossed over $200 million, studios have realized that horror films can make them a lot of money with very little risk. Some criticize this trend because studios can continue to roll out sequels to movies like Paranormal Activity (the most profitable movie of all time by most reports) and The Purge even after they are no longer good. They cost very little to make and their brand recognition means they will likely make a decent run at the box office. However, the low financial risk of these films also gives studios the opportunity to give new filmmakers more freedom. A studio producing a big budget film would never hand over the reins to a newcomer without overseeing every step of the production carefully. Because of the low risk, Jordan Peele, a first-time director known for a comedy sketch show, had the freedom to do what he wants with his first movie. The result of this first-time, unproven director getting directorial freedom is Get Out, one of the most critically-acclaimed movies of 2017. This trend will invariably lead to more lazy horror sequels, but it could also lead to the de-stigmatization of the horror genre. Jordan Peele received an Oscar for Original Screenplay for Get Out; is a Best Picture-winning horror film far off?
 

Part B

Which sentence from the passage best supports the answer to Part A?

A
“Many critics trace the modern summer blockbuster back to Jaws in 1975, when director Steven Spielberg and Universal deliberately delayed the opening of their film to coincide with summer.”
B
“However, if you do some research on the most profitable movies of all time, none of the films cost even $1 million to make.”
C
“If you do some research on the highest grossing movies of all time, the list is exclusively big-budget movies.”
D
“Spielberg’s friend, George Lucas, released each of the films in his original Star Wars trilogy during the summer.”
Question 25 Explanation: 
The correct answer is answer choice (C). While this sentence does not explicitly state that the rewards for a big budget movie can be tremendous, the fact that all of the highest grossing movies of all time are big budget films supports the statement from Part A. None of the other choices speak to the risk or reward inherent in making these films.
Question 26
The Modern Horror Movie

(1) The American Film Industry, like any business market, has a history of adapting itself to meet the demands of its customers. When viewers respond to a film trend, movies studios do their best to capitalize on that trend. This can sometimes lead to disastrous losses, but it has also helped to shape American films throughout the history of the industry. Summer blockbusters, for example, have become an annual tradition. Up until the last few years, studios almost exclusively released their big-budget films during the summer (ticket sales have recently proven that these big budget films can flourish during other seasons as well).

(2) Summer wasn’t always the time for big budget movies. In fact, summer used to be the slow season for movie studios. People were out at the beach all day and didn’t want to spend their evenings inside of a movie theater. Many critics trace the modern summer blockbuster back to Jaws in 1975, when director Steven Spielberg and Universal deliberately delayed the opening of their film to coincide with summer. Since Jaws is a film about a shark terrorizing a sleepy beach town, Spielberg and Universal figured it would have the most impact during the season when people frequented the beach. They were right. After Jaws, more movies began dipping their toes into the summer market. Spielberg’s friend, George Lucas, released each of the films in his original Star Wars trilogy during the summer. By the 90’s big-budget summer blockbusters were an annual tradition.

(3) In the last 20 years, however, we have seen a trend in films that is almost the antithesis of the big-budget summer blockbusters. Studios have proven through the last 40 years that big budget movies can pay big dividends. In the last 20 years, though, some movie studios are cashing in on the fact that small budget movies can have even better returns. A contemporary big-budget blockbuster can cost around $250 million to make, and another $250 million to properly market. If that film makes $1 billion, then the movie studio has made $500 million, or 100% profit. That is an amazing return on the studio’s investment. However, if that same studio made a smaller film for $5 million, and spent another $5 million on marketing, they could reach the same percentage of return on their investment if the movie makes $10 million. The justification behind big budget movies is that betting big potentially gives the studio the biggest payoff. The downfall, however, is that the studios are shelling out a lot of money up front; if the movie doesn’t do great, it’s a failure.

(4) If you do some research on the highest grossing movies of all time, the list is exclusively big-budget movies. The top five movies on the list, Avatar, Titanic, Star Wars: The Force Awakens, Avengers: Infinity War, and Jurassic World cost over $1 billion to be made (not counting the money spent on marketing). However, if you do some research on the most profitable movies of all time, none of the films on the list cost even $1 million to make.

(5) The studios that have embraced this low-budget, high return trend the most are studios who produce horror movies. Since the release of The Blair Witch Project in 1999, a film that cost $600,000 to make, and grossed over $200 million, studios have realized that horror films can make them a lot of money with very little risk. Some criticize this trend because studios can continue to roll out sequels to movies like Paranormal Activity (the most profitable movie of all time by most reports) and The Purge even after they are no longer good. They cost very little to make and their brand recognition means they will likely make a decent run at the box office. However, the low financial risk of these films also gives studios the opportunity to give new filmmakers more freedom. A studio producing a big budget film would never hand over the reins to a newcomer without overseeing every step of the production carefully. Because of the low risk, Jordan Peele, a first-time director known for a comedy sketch show, had the freedom to do what he wants with his first movie. The result of this first-time, unproven director getting directorial freedom is Get Out, one of the most critically-acclaimed movies of 2017. This trend will invariably lead to more lazy horror sequels, but it could also lead to the de-stigmatization of the horror genre. Jordan Peele received an Oscar for Original Screenplay for Get Out; is a Best Picture-winning horror film far off?
 

Read these two sentences from the passage.

This trend will invariably lead to more lazy horror sequels, but it could also lead to the de-stigmatization of the horror genre. Jordan Peele received an Oscar for Original Screenplay for Get Out; is a Best Picture-winning horror film far off?

Which of these words best explains the meaning of stigmatization in the sentences above?

A
Acceptance
B
Hatred
C
Condemnation
D
Apathy
Question 26 Explanation: 
The correct answer is answer choice (C). The writer is positing that this trend might lead to more critical acceptance of horror films. Removing the stigma (de-stigmatization) would require critics to stop their condemnation of horror films. Answer choice (A) is an antonym of stigmatization, and answer choices (B) and (C) do not quite capture the emotion the writer wants to convey.
Question 27
The Modern Horror Movie

(1) The American Film Industry, like any business market, has a history of adapting itself to meet the demands of its customers. When viewers respond to a film trend, movies studios do their best to capitalize on that trend. This can sometimes lead to disastrous losses, but it has also helped to shape American films throughout the history of the industry. Summer blockbusters, for example, have become an annual tradition. Up until the last few years, studios almost exclusively released their big-budget films during the summer (ticket sales have recently proven that these big budget films can flourish during other seasons as well).

(2) Summer wasn’t always the time for big budget movies. In fact, summer used to be the slow season for movie studios. People were out at the beach all day and didn’t want to spend their evenings inside of a movie theater. Many critics trace the modern summer blockbuster back to Jaws in 1975, when director Steven Spielberg and Universal deliberately delayed the opening of their film to coincide with summer. Since Jaws is a film about a shark terrorizing a sleepy beach town, Spielberg and Universal figured it would have the most impact during the season when people frequented the beach. They were right. After Jaws, more movies began dipping their toes into the summer market. Spielberg’s friend, George Lucas, released each of the films in his original Star Wars trilogy during the summer. By the 90’s big-budget summer blockbusters were an annual tradition.

(3) In the last 20 years, however, we have seen a trend in films that is almost the antithesis of the big-budget summer blockbusters. Studios have proven through the last 40 years that big budget movies can pay big dividends. In the last 20 years, though, some movie studios are cashing in on the fact that small budget movies can have even better returns. A contemporary big-budget blockbuster can cost around $250 million to make, and another $250 million to properly market. If that film makes $1 billion, then the movie studio has made $500 million, or 100% profit. That is an amazing return on the studio’s investment. However, if that same studio made a smaller film for $5 million, and spent another $5 million on marketing, they could reach the same percentage of return on their investment if the movie makes $10 million. The justification behind big budget movies is that betting big potentially gives the studio the biggest payoff. The downfall, however, is that the studios are shelling out a lot of money up front; if the movie doesn’t do great, it’s a failure.

(4) If you do some research on the highest grossing movies of all time, the list is exclusively big-budget movies. The top five movies on the list, Avatar, Titanic, Star Wars: The Force Awakens, Avengers: Infinity War, and Jurassic World cost over $1 billion to be made (not counting the money spent on marketing). However, if you do some research on the most profitable movies of all time, none of the films on the list cost even $1 million to make.

(5) The studios that have embraced this low-budget, high return trend the most are studios who produce horror movies. Since the release of The Blair Witch Project in 1999, a film that cost $600,000 to make, and grossed over $200 million, studios have realized that horror films can make them a lot of money with very little risk. Some criticize this trend because studios can continue to roll out sequels to movies like Paranormal Activity (the most profitable movie of all time by most reports) and The Purge even after they are no longer good. They cost very little to make and their brand recognition means they will likely make a decent run at the box office. However, the low financial risk of these films also gives studios the opportunity to give new filmmakers more freedom. A studio producing a big budget film would never hand over the reins to a newcomer without overseeing every step of the production carefully. Because of the low risk, Jordan Peele, a first-time director known for a comedy sketch show, had the freedom to do what he wants with his first movie. The result of this first-time, unproven director getting directorial freedom is Get Out, one of the most critically-acclaimed movies of 2017. This trend will invariably lead to more lazy horror sequels, but it could also lead to the de-stigmatization of the horror genre. Jordan Peele received an Oscar for Original Screenplay for Get Out; is a Best Picture-winning horror film far off?
 

According to the passage, why do some movie insiders consider Paranormal Activity to be the most successful film of all time?

A
It made the biggest profit based on how much it cost to make.
B
It made more money than any other film.
C
It won a lot of movie awards.
D
It helped pave the way for movies like Get Out.
Question 27 Explanation: 
The correct answer is answer choice (A). The writer notes that “Paranormal Activity [is] (the most profitable movie of all time by most reports)” as he is discussing the recent success of low-budget films. Answer choice (D) is tempting because, based on the passage, the writer would agree with what it is saying. However, the writer does not suggest that its influence on Get Out is what makes it a successful film. Answer choices (B) and (C) are directly refuted in the passage and absent from the passage, respectively.
Question 28
The Modern Horror Movie

(1) The American Film Industry, like any business market, has a history of adapting itself to meet the demands of its customers. When viewers respond to a film trend, movies studios do their best to capitalize on that trend. This can sometimes lead to disastrous losses, but it has also helped to shape American films throughout the history of the industry. Summer blockbusters, for example, have become an annual tradition. Up until the last few years, studios almost exclusively released their big-budget films during the summer (ticket sales have recently proven that these big budget films can flourish during other seasons as well).

(2) Summer wasn’t always the time for big budget movies. In fact, summer used to be the slow season for movie studios. People were out at the beach all day and didn’t want to spend their evenings inside of a movie theater. Many critics trace the modern summer blockbuster back to Jaws in 1975, when director Steven Spielberg and Universal deliberately delayed the opening of their film to coincide with summer. Since Jaws is a film about a shark terrorizing a sleepy beach town, Spielberg and Universal figured it would have the most impact during the season when people frequented the beach. They were right. After Jaws, more movies began dipping their toes into the summer market. Spielberg’s friend, George Lucas, released each of the films in his original Star Wars trilogy during the summer. By the 90’s big-budget summer blockbusters were an annual tradition.

(3) In the last 20 years, however, we have seen a trend in films that is almost the antithesis of the big-budget summer blockbusters. Studios have proven through the last 40 years that big budget movies can pay big dividends. In the last 20 years, though, some movie studios are cashing in on the fact that small budget movies can have even better returns. A contemporary big-budget blockbuster can cost around $250 million to make, and another $250 million to properly market. If that film makes $1 billion, then the movie studio has made $500 million, or 100% profit. That is an amazing return on the studio’s investment. However, if that same studio made a smaller film for $5 million, and spent another $5 million on marketing, they could reach the same percentage of return on their investment if the movie makes $10 million. The justification behind big budget movies is that betting big potentially gives the studio the biggest payoff. The downfall, however, is that the studios are shelling out a lot of money up front; if the movie doesn’t do great, it’s a failure.

(4) If you do some research on the highest grossing movies of all time, the list is exclusively big-budget movies. The top five movies on the list, Avatar, Titanic, Star Wars: The Force Awakens, Avengers: Infinity War, and Jurassic World cost over $1 billion to be made (not counting the money spent on marketing). However, if you do some research on the most profitable movies of all time, none of the films on the list cost even $1 million to make.

(5) The studios that have embraced this low-budget, high return trend the most are studios who produce horror movies. Since the release of The Blair Witch Project in 1999, a film that cost $600,000 to make, and grossed over $200 million, studios have realized that horror films can make them a lot of money with very little risk. Some criticize this trend because studios can continue to roll out sequels to movies like Paranormal Activity (the most profitable movie of all time by most reports) and The Purge even after they are no longer good. They cost very little to make and their brand recognition means they will likely make a decent run at the box office. However, the low financial risk of these films also gives studios the opportunity to give new filmmakers more freedom. A studio producing a big budget film would never hand over the reins to a newcomer without overseeing every step of the production carefully. Because of the low risk, Jordan Peele, a first-time director known for a comedy sketch show, had the freedom to do what he wants with his first movie. The result of this first-time, unproven director getting directorial freedom is Get Out, one of the most critically-acclaimed movies of 2017. This trend will invariably lead to more lazy horror sequels, but it could also lead to the de-stigmatization of the horror genre. Jordan Peele received an Oscar for Original Screenplay for Get Out; is a Best Picture-winning horror film far off?
 

According to the passage, why does the current trend of low-budget horror films lead to more bad sequels?

A
Studios still don’t understand how to evaluate what films are good and what films are bad.
B
They don’t cost much to make and people go see them because of brand recognition.
C
When studios lose money on one movie, they will often double-down and try to make their money back with a sequel.
D
Studios are giving creative license to too many new directors.
Question 28 Explanation: 
The correct answer is answer choice (B). In discussing these sequels, the writer writes, “They cost very little to make and their brand recognition means they will likely make a decent run at the box office.” Answer choices (A) and (C) are not explored in the passage, and the writer applauds the creative freedom studios are giving new directors, which makes answer choice (D) incorrect.
Question 29
The Modern Horror Movie

(1) The American Film Industry, like any business market, has a history of adapting itself to meet the demands of its customers. When viewers respond to a film trend, movies studios do their best to capitalize on that trend. This can sometimes lead to disastrous losses, but it has also helped to shape American films throughout the history of the industry. Summer blockbusters, for example, have become an annual tradition. Up until the last few years, studios almost exclusively released their big-budget films during the summer (ticket sales have recently proven that these big budget films can flourish during other seasons as well).

(2) Summer wasn’t always the time for big budget movies. In fact, summer used to be the slow season for movie studios. People were out at the beach all day and didn’t want to spend their evenings inside of a movie theater. Many critics trace the modern summer blockbuster back to Jaws in 1975, when director Steven Spielberg and Universal deliberately delayed the opening of their film to coincide with summer. Since Jaws is a film about a shark terrorizing a sleepy beach town, Spielberg and Universal figured it would have the most impact during the season when people frequented the beach. They were right. After Jaws, more movies began dipping their toes into the summer market. Spielberg’s friend, George Lucas, released each of the films in his original Star Wars trilogy during the summer. By the 90’s big-budget summer blockbusters were an annual tradition.

(3) In the last 20 years, however, we have seen a trend in films that is almost the antithesis of the big-budget summer blockbusters. Studios have proven through the last 40 years that big budget movies can pay big dividends. In the last 20 years, though, some movie studios are cashing in on the fact that small budget movies can have even better returns. A contemporary big-budget blockbuster can cost around $250 million to make, and another $250 million to properly market. If that film makes $1 billion, then the movie studio has made $500 million, or 100% profit. That is an amazing return on the studio’s investment. However, if that same studio made a smaller film for $5 million, and spent another $5 million on marketing, they could reach the same percentage of return on their investment if the movie makes $10 million. The justification behind big budget movies is that betting big potentially gives the studio the biggest payoff. The downfall, however, is that the studios are shelling out a lot of money up front; if the movie doesn’t do great, it’s a failure.

(4) If you do some research on the highest grossing movies of all time, the list is exclusively big-budget movies. The top five movies on the list, Avatar, Titanic, Star Wars: The Force Awakens, Avengers: Infinity War, and Jurassic World cost over $1 billion to be made (not counting the money spent on marketing). However, if you do some research on the most profitable movies of all time, none of the films on the list cost even $1 million to make.

(5) The studios that have embraced this low-budget, high return trend the most are studios who produce horror movies. Since the release of The Blair Witch Project in 1999, a film that cost $600,000 to make, and grossed over $200 million, studios have realized that horror films can make them a lot of money with very little risk. Some criticize this trend because studios can continue to roll out sequels to movies like Paranormal Activity (the most profitable movie of all time by most reports) and The Purge even after they are no longer good. They cost very little to make and their brand recognition means they will likely make a decent run at the box office. However, the low financial risk of these films also gives studios the opportunity to give new filmmakers more freedom. A studio producing a big budget film would never hand over the reins to a newcomer without overseeing every step of the production carefully. Because of the low risk, Jordan Peele, a first-time director known for a comedy sketch show, had the freedom to do what he wants with his first movie. The result of this first-time, unproven director getting directorial freedom is Get Out, one of the most critically-acclaimed movies of 2017. This trend will invariably lead to more lazy horror sequels, but it could also lead to the de-stigmatization of the horror genre. Jordan Peele received an Oscar for Original Screenplay for Get Out; is a Best Picture-winning horror film far off?
 

According to the passage, how are Jaws and the three original Star Wars films similar?

A
They are all in the top 5 of the highest grossing films of all time.
B
They are all low-budget horror films.
C
They are all directed by George Lucas.
D
They were all released during the summer.
Question 29 Explanation: 
The correct answer is answer choice (D). The writer introduces how Jaws as the first summer blockbuster, then discusses how George Lucas followed the trend and released the each other the original Star Wars films in the summer. The passage directly refutes the other three answer choices.
Question 30
The Modern Horror Movie

(1) The American Film Industry, like any business market, has a history of adapting itself to meet the demands of its customers. When viewers respond to a film trend, movies studios do their best to capitalize on that trend. This can sometimes lead to disastrous losses, but it has also helped to shape American films throughout the history of the industry. Summer blockbusters, for example, have become an annual tradition. Up until the last few years, studios almost exclusively released their big-budget films during the summer (ticket sales have recently proven that these big budget films can flourish during other seasons as well).

(2) Summer wasn’t always the time for big budget movies. In fact, summer used to be the slow season for movie studios. People were out at the beach all day and didn’t want to spend their evenings inside of a movie theater. Many critics trace the modern summer blockbuster back to Jaws in 1975, when director Steven Spielberg and Universal deliberately delayed the opening of their film to coincide with summer. Since Jaws is a film about a shark terrorizing a sleepy beach town, Spielberg and Universal figured it would have the most impact during the season when people frequented the beach. They were right. After Jaws, more movies began dipping their toes into the summer market. Spielberg’s friend, George Lucas, released each of the films in his original Star Wars trilogy during the summer. By the 90’s big-budget summer blockbusters were an annual tradition.

(3) In the last 20 years, however, we have seen a trend in films that is almost the antithesis of the big-budget summer blockbusters. Studios have proven through the last 40 years that big budget movies can pay big dividends. In the last 20 years, though, some movie studios are cashing in on the fact that small budget movies can have even better returns. A contemporary big-budget blockbuster can cost around $250 million to make, and another $250 million to properly market. If that film makes $1 billion, then the movie studio has made $500 million, or 100% profit. That is an amazing return on the studio’s investment. However, if that same studio made a smaller film for $5 million, and spent another $5 million on marketing, they could reach the same percentage of return on their investment if the movie makes $10 million. The justification behind big budget movies is that betting big potentially gives the studio the biggest payoff. The downfall, however, is that the studios are shelling out a lot of money up front; if the movie doesn’t do great, it’s a failure.

(4) If you do some research on the highest grossing movies of all time, the list is exclusively big-budget movies. The top five movies on the list, Avatar, Titanic, Star Wars: The Force Awakens, Avengers: Infinity War, and Jurassic World cost over $1 billion to be made (not counting the money spent on marketing). However, if you do some research on the most profitable movies of all time, none of the films on the list cost even $1 million to make.

(5) The studios that have embraced this low-budget, high return trend the most are studios who produce horror movies. Since the release of The Blair Witch Project in 1999, a film that cost $600,000 to make, and grossed over $200 million, studios have realized that horror films can make them a lot of money with very little risk. Some criticize this trend because studios can continue to roll out sequels to movies like Paranormal Activity (the most profitable movie of all time by most reports) and The Purge even after they are no longer good. They cost very little to make and their brand recognition means they will likely make a decent run at the box office. However, the low financial risk of these films also gives studios the opportunity to give new filmmakers more freedom. A studio producing a big budget film would never hand over the reins to a newcomer without overseeing every step of the production carefully. Because of the low risk, Jordan Peele, a first-time director known for a comedy sketch show, had the freedom to do what he wants with his first movie. The result of this first-time, unproven director getting directorial freedom is Get Out, one of the most critically-acclaimed movies of 2017. This trend will invariably lead to more lazy horror sequels, but it could also lead to the de-stigmatization of the horror genre. Jordan Peele received an Oscar for Original Screenplay for Get Out; is a Best Picture-winning horror film far off?
 

What purpose does the writer’s use of specific dollar amounts in paragraphs 3, 4, and 5 serve in the passage?

A
They are meant to confuse the reader and conceal the writer’s main point.
B
They provide evidence of the writer’s support of blockbusters over low-budget films.
C
They illustrate his point that low-budget films are sometimes more successful than high-budget films.
D
They are designed to persuade the reader to go see a low-budget horror film instead of a summer blockbuster.
Question 30 Explanation: 
The correct answer is answer choice (C). The writer uses specific numbers to show that while a low budget movie like Paranormal Activity might make less money than a movie like Avatar, it is more successful when you consider its rate of return. Answer choice (A) is incorrect because the writer does not want to confuse his readers, and answer choices (B) and (D) are incorrect because the writer gives no suggestion that favors one type of film over the other.
Once you are finished, click the button below. Any items you have not completed will be marked incorrect. Get Results
There are 30 questions to complete.
List
Return
Shaded items are complete.
12345
678910
1112131415
1617181920
2122232425
2627282930
End
Return